Đến nội dung

Gachdptrai12 nội dung

Có 274 mục bởi Gachdptrai12 (Tìm giới hạn từ 26-04-2020)



Sắp theo                Sắp xếp  

#638208 VMF's Marathon Bất Đẳng Thức Olympic

Đã gửi bởi Gachdptrai12 on 05-06-2016 - 10:13 trong Bất đẳng thức và cực trị

Bài 25.

Giả sử $a\geq b\geq c,$

$\frac{4(a^2+b^2+c^2)}{ab+bc+ca}+\sqrt{3}\sum_{cyc}\frac{a+2b}{\sqrt{a^2+2b^2}}-13$
$=\sum_{cyc}\left ( \frac{1}{ab+bc+ca}-\frac{3}{(b+2c+\sqrt{3(b^2+2c^2)})\sqrt{3(b^2+2c^2)}} \right )(b-c)^2$
$\geq \left ( \frac{1}{ab+bc+ca}-\frac{3}{(c+2a+\sqrt{3(c^2+2a^2)})\sqrt{3(c^2+2a^2)}} \right )(c-a)^2+\left ( \frac{1}{ab+bc+ca}-\frac{3}{(b+2c+\sqrt{3(b^2+2c^2)})\sqrt{3(b^2+2c^2)}} \right )(b-c)^2$
$\geq \frac{1}{b^2}\left ( \frac{a^2}{ab+bc+ca}-\frac{3a^2}{(c+2a+\sqrt{3(c^2+2a^2)})\sqrt{3(c^2+2a^2)}}+\frac{b^2}{ab+bc+ca}-\frac{3b^2}{(b+2c+\sqrt{3(b^2+2c^2)})\sqrt{3(b^2+2c^2)}} \right )(b-c)^2$
$\geq \frac{1}{b^2}\left ( \frac{a^2+b^2}{ab+bc+ca}-\frac{3a^2}{2(c+2a)^2}-\frac{3b^2}{2(b+2c)^2} \right )(b-c)^2$
$\geq \frac{(8a^4b^2+8a^2b^4-15a^3b^3)+(19a^4c^2-19a^3b^2c)+(4a^4bc-4a^2bc^3)}{2b^2(ab+bc+ca)(c+2a)^2(b+2c)^2}\geq 0\blacksquare$

 Bách coi lại bài này đâu thể giả sử $a\geq b\geq c$ và cái tô đỏ thứ 2 là chưa chắc lớn hơn




#638087 VMF's Marathon Bất Đẳng Thức Olympic

Đã gửi bởi Gachdptrai12 on 04-06-2016 - 21:18 trong Bất đẳng thức và cực trị

Bài toán 25. (Võ Quốc Bá Cẩn) Cho $a,b,c$ là các số thực không âm, chứng minh

$\dfrac{4(a^{2}+b^{2}+c^{2})}{ab+bc+ca}+\sqrt{3}\left(\frac{a+2b}{\sqrt{a^{2}+2b^{2}}}+\frac{b+2c}{\sqrt{b^{2}+2c^{2}}}+\frac{c+2a}{\sqrt{c^{2}+2a^{2}}}\right)\geq 13$




#638079 VMF's Marathon Bất Đẳng Thức Olympic

Đã gửi bởi Gachdptrai12 on 04-06-2016 - 21:07 trong Bất đẳng thức và cực trị

Vì không thấy ai đăng tiếp, mình xin góp một bài, xin lấy tiêu đề là bài toán 24, nếu các bạn không đồng ý thì có thể bỏ bài toán này!

 

Bài toán 24: (Nguồn: Juliel)

Cho các số thực dương a,b,c. Chứng minh:

$$\sqrt[7]{\frac{a^{7}+b^{7}}{2}}+\sqrt[7]{\frac{b^{7}+c^{7}}{2}}+\sqrt[7]{\frac{c^{7}+a^{7}}{2}}\leq \left ( a+b+c \right )^{10}\left ( \frac{1}{9a}+\frac{1}{9b}+\frac{1}{9c} \right )^{9}$$

 

Áp dụng BĐT AM-GM :

$\dfrac{a^7+b^7}{(a+b)^6}+\dfrac{a+b}{2^6}+\dfrac{a+b}{2^6}+\dfrac{a+b}{2^6}+\dfrac{a+b}{2^6}+\dfrac{a+b}{2^6}+\dfrac{a+b}{2^6}\geq 7\sqrt[7]{\dfrac{a^7+b^7}{(2^6)^6}}=\dfrac{7}{32}\sqrt[7]{\dfrac{a^7+b^7}{2}}$

Tức là :
$\dfrac{a^7+b^7}{(a+b)^6}+\dfrac{3}{32}(a+b)\geq \dfrac{7}{32}\sqrt[7]{\dfrac{a^7+b^7}{2}}$
Ta có :
$(a+b)^7=a^7+b^7+7ab(a^5+b^5)+21a^2b^2(a^3+b^3)+35a^3b^3(a+b)=a^7+b^7+7ab\left [ a^5+b^5+3ab(a^3+b^3)+5a^2b^2(a+b) \right ]$
Theo AM-GM :
$12a^5+9b^5+7a^4b+2ab^4\geq 30\sqrt[30]{(a^5)^{12}.(b^5)^9.(a^4b)^7.(ab^4)^2}=30a^3b$
$12b^5+9a^5+7b^4a+2ba^4\geq 30\sqrt[30]{(b^5)^{12}.(a^5)^9.(b^4a)^7.(ba^4)^2}=30b^3a$
Cộng vế theo vế hai kết quả trên thì được :
$7(a^5+b^5)+3ab(a^3+b^3)\geq 10a^2b^2(a+b)$
Suy ra :
 
$16(a^5+b^5)+48ab(a^3+b^3)+80a^2b^2(a+b)\geq 9(a^5+b^5)+45ab(a^3+b^3)+90a^2b^2(a+b)\Leftrightarrow 16\left [ a^5+b^5+3ab(a^3+b^3)+5a^2b^2(a+b) \right ]\geq 9\left [ a^5+b^5+5ab(a^3+b^3)+10a^2b^2(a+b) \right ]\Leftrightarrow a^5+b^5+3ab(a^3+b^3)+5a^2b^2(a+b) \geq \dfrac{9}{16}(a+b)^5$
Từ đó ta được :
 
$(a+b)^7\geq a^7+b^7+7ab.\dfrac{9}{16}(a+b)^5\Leftrightarrow \dfrac{a^7+b^7}{(a+b)^6}\leq a+b-\dfrac{63ab}{16(a+b)}$
Lại được tiếp :
$\dfrac{7}{32}\sqrt[7]{\dfrac{a^7+b^7}{2}}\leq \dfrac{3}{32}(a+b)+(a+b)-\dfrac{63ab}{16(a+b)}=\dfrac{35}{32}(a+b)-\dfrac{63ab}{16(a+b)}\Leftrightarrow \sqrt[7]{\dfrac{a^7+b^7}{2}}\leq 5(a+b)-\dfrac{18ab}{a+b}$
Thiết lập các BĐT tương tự rồi cộng chúng vế theo vế, ta dẫn đến việc chứng minh :
$\left ( a+b+c \right )^{10}\left ( \dfrac{1}{9a}+\dfrac{1}{9b} +\dfrac{1}{9c}\right )^9+\dfrac{18ab}{a+b}+\dfrac{18bc}{b+c}+\dfrac{18ca}{c+a}\geq 10(a+b+c)\Leftrightarrow \dfrac{(a+b+c)^{10}.(ab+bc+ca)^9}{(abc)^9.9^9}+\dfrac{18}{\dfrac{1}{a}+\dfrac{1}{b}}+\dfrac{18}{\dfrac{1}{b}+\dfrac{1}{c}}+\dfrac{18}{\dfrac{1}{c}+\dfrac{1}{a}}\geq 10(a+b+c)$
Theo Cauchy-Schwarz :
$\dfrac{18}{\dfrac{1}{a}+\dfrac{1}{b}}+\dfrac{18}{\dfrac{1}{b}+\dfrac{1}{c}}+\dfrac{18}{\dfrac{1}{c}+\dfrac{1}{a}}\geq \dfrac{18.9}{2\left ( \dfrac{1}{a}+\dfrac{1}{b} +\dfrac{1}{c}\right )}=\dfrac{81abc}{ab+bc+ca}$
Như vậy cần chỉ ra :
$\dfrac{(a+b+c)^{10}.(ab+bc+ca)^9}{(abc)^9.9^9}+\dfrac{81abc}{ab+bc+ca}\geq 10(a+b+c)$
Nhưng điều này hiển nhiên đúng theo AM-GM cho 10 số :
$\dfrac{(a+b+c)^{10}.(ab+bc+ca)^9}{(abc)^9.9^9}+\dfrac{81abc}{ab+bc+ca}=\dfrac{(a+b+c)^{10}.(ab+bc+ca)^9}{(abc)^9.9^9}+9.\dfrac{9abc}{ab+bc+ca}\geq 10\sqrt[10]{\dfrac{(a+b+c)^{10}.(ab+bc+ca)^9}{(abc)^9.9^9}.\dfrac{(9abc)^9}{(ab+bc+ca)^9}}=10(a+b+c)$
p/s:đề nghị sửa đề  :D  :D



#637923 VMF's Marathon Bất Đẳng Thức Olympic

Đã gửi bởi Gachdptrai12 on 03-06-2016 - 22:25 trong Bất đẳng thức và cực trị

Lời giải này ý tưởng đúng nhưng gõ sai nhiều chỗ, em nên viết nghiêm túc hơn. Ý tưởng lời giải anh nói ở trên là quy về chứng minh bất đẳng thức $1$ biến:

$$ x^3y+y^3z+z^3x \leq y(x^3+z^3+xyz) \leq y(x^2+z^2+2xyz)^{\frac{3}{2}}=y(1-y^2)^{\frac{3}{2}} \leq \frac{3\sqrt{3}}{16}.$$
Phần chứng minh $x^3+z^3+xyz\leq (x^2+z^2+2xyz)^{\frac{3}{2}}$ cơ bản là giống với lời giải của em. Chú ý đây là bất đẳng thức hoán vị nên không dùng dồn biến kiểu quy về chứng minh $f(\sqrt{\frac{1-z}{2}},\sqrt{\frac{1-z}{2}},z)\leq \frac{3\sqrt{3}}{16}$ như em nói được. 

 

chắc có lẽ cách em sai 




#637828 chứng minh GK và HL cắt nhau trên AO

Đã gửi bởi Gachdptrai12 on 03-06-2016 - 16:36 trong Hình học

cho tam giác ABC nội tiếp (O).Đường cao AD.(A) là đường trong bất kì tâm A.E,F bất kì thuộc A sao cho E,F đối xứng với nhau qua AD và tia AE nằm giữa AB và AF.Trên (A) lấy G,H sao cho GH vuông góc OA đồng thời tia AB nằm giữa tia AG,AC.CE,BF giao (A) tại P,Q khác E,F.(BPG) và (CQH) lần lượt cắt  BA,CA tại K,L khác B,C.chứng minh GK và HL cắt nhau trên AO




#637814 VMF's Marathon Bất Đẳng Thức Olympic

Đã gửi bởi Gachdptrai12 on 03-06-2016 - 16:03 trong Bất đẳng thức và cực trị

 Bài toán 23. (Sưu tầm) Cho các số thực không âm $x,y,z$ thỏa mãn $x^2+y^2+z^2+2xyz=1$. Chứng minh rằng

\[x^3y+y^3z+z^3x\leq \dfrac{3\sqrt{3}}{16}\]

ta chỉ cần chứng minh bdt khi $x\geq y\geq z$ 

ta có $x\geq \frac{1}{2}\Rightarrow 1=x^{2}+y^{2}+z^{2}+2xyz\geq x^{2}+y^{2}+z^{2}+yz$

từ đó ta có $x^{3}y+y^{3}z+z^{3}x\leq xy(x^{2}+xy+y^{2})=S$

mà theo bddt AM-GM ta có

$S^{2}=x^{2}y^{2}(x^{2}+zy+z^{2})^{2}=27\frac{x^{2}}{3}y^{2}(\frac{x^{2}+yz+z^{2}}{3})^{2}\leq \frac{27}{256}(\frac{x^{2}}{3}+\frac{2(x^{2}+yz+z^{2})}{3}+y^{2})^{4}= \frac{27}{256}(x^{2}+y^{2}+\frac{2(yz+z^{2})}{3})^{4}\leq \frac{27}{256}(x^{2}+y^{2}+z^{2}+yz)^{4}\leq \frac{27}{256}\Rightarrow S\leq \frac{3\sqrt{3}}{16}$

vậy ta có đpcm

 

Về cơ bản thì bài toán này có thể dử dụng ý tưởng như bài  toán 19, 20. Tất nhiên bài này lời giải khá đơn giản vì chỉ có 1 dấu đẳng thức xảy ra (không tính hoán vị).

em nghĩ nó không dễ đâu ạ dù chỉ có 1 dấu bằng nhưng rất khó đoán phải dùng máy tính :3 :v

em đang suy nghĩ 1 cách khác đó là chứng minh $f(\sqrt{\frac{1-z}{2}},\sqrt{\frac{1-z}{2}},z)\geq \frac{3\sqrt{3}}{16}$




#637156 VMF's Marathon Bất Đẳng Thức Olympic

Đã gửi bởi Gachdptrai12 on 31-05-2016 - 16:37 trong Bất đẳng thức và cực trị

Lời giải bài 21. Lời giải cuả mình phần đầu giống anh Huyện nên mình bỏ phần đầu nhé
Ta chứng minh
$\sum \frac{5}{2a^2+\sqrt{4a^4+5abc}}\ge 3$
Tuy nhiên áp dụng C-S
${\sum \frac{5}{2a^2+\sqrt{4a^4+5abc}}=\sum \frac{5b^2c^2}{2a^2b^2c^2+b^2c^2\sqrt{4a^4+5abc}}\ge \frac{5(ab+bc+ca)^2}{6a^2b^2c^2+\sum b^2c^2\sqrt{4a^4+5abc}}}$
Ta sẽ chứng minh
$5(\frac{1}{a}+\frac{1}{b}+\frac{1}{c})^2\ge 18+\sum 3\frac{\sqrt{4a^4+5abc}}{a^2}$
Áp dụng AM-GM
$3\frac{\sqrt{4a^4+5abc}}{a^2}\le \frac{9+4a^4+5abc}{2a^2}$
Ta chứng minh$\sum \frac{1}{2a^2}+10\sum \frac{1}{ab}\ge 2\sum a^2+3+\frac{5}{2}\sum \frac{1}{a}\sum ab$
Là 1 bất đẳng thức đúng khi $a+b+c=3$




#636647 VMF's Marathon Bất Đẳng Thức Olympic

Đã gửi bởi Gachdptrai12 on 29-05-2016 - 21:45 trong Bất đẳng thức và cực trị

P/s. Anh nghĩ lời giải của Long không đúng, bài này chắc có kiểu dồn biến theo kiểu thừa trừ của anh Cẩn. Còn điều kiện của bài 17dương hay không âm nhỉ ?

lời giải của Long em nghĩ nó đúng mà với lại đó là dồn biến thừa trừ luôn mà anh Huyện




#636646 VMF's Marathon Bất Đẳng Thức Olympic

Đã gửi bởi Gachdptrai12 on 29-05-2016 - 21:39 trong Bất đẳng thức và cực trị

 Bài toán 17. (Nguyễn Quốc Anh) Cho các số thực dương $x,y,z$ thỏa mãn $x+y+z=32$. Tìm giá trị lớn nhất của

$P=x^3y+y^3z+z^3x$

 Lời giải : Giả sử $x=\max \{x,y,z\}$

Ta có

$a^{3}b+b^{3}c+c^{3}a\leq a^{3}b+a^{2}bc+a^{2}c^{2}\leq a^{3}b+a^{2}bc+\frac{a^{2}c(a+c)}{2}=a^{2}(a+c)\left(b+\frac{c}{2}\right)$

Áp dụng AM-GM ta có

$a^{2}(a+c)\left(b+\frac{c}{2}\right)=27\frac{a^{2}(a+c)}{27}\left(b+\frac{c}{2}\right)\leq \frac{27}{256}\left(\frac{a}{3}+\frac{a}{3}+\frac{a+c}{3}+b+\frac{c}{2}\right)^{4}= \frac{27}{256}\left(a+b+\frac{5c}{6}\right)^{4}\leq \frac{27}{256}(a+b+c)^{4}=\frac{32^{4}.27}{256}$




#636326 VMF's Marathon Bất Đẳng Thức Olympic

Đã gửi bởi Gachdptrai12 on 28-05-2016 - 20:11 trong Bất đẳng thức và cực trị

Bài 15. Cho $a,b,c>0$ sao cho $ab+bc+ca=1$. Chứng minh
$$(a^2+ab+b^2)(b^2+bc+c^2)(c^2+ca+a^2)\ge\frac{2}{3}(\frac{a}{b+c}+\frac{b}{c+a}+\frac{c}{a+b}).$$

Lời giải bài 15

Ta sẽ đồng bậc hóa bất đẳng thức

$\Leftrightarrow(a^{2}+ab+b^{2})(b^{2}+bc+c^{2})(c^{2}+ca+a^{2})\ge\frac{2}{3}(\frac{a}{b+c}+\frac{b}{c+a}+\frac{c}{a+b})(ab+bc+ca)^{3}$
Bây giờ ta sẽ chứng minh
$2(\sum \frac{a}{b+c})\leq \frac{2(a^{2}+b^{2}+c^{2})+ab+bc+ca}{ab+bc+ca}$

$\Leftrightarrow 2(\sum \frac{a}{b+c})-3\geq \frac{2(a^{2}+b^{2}+c^{2}-ab-bc-ca)}{ab+bc+ca}$

$\Leftrightarrow \sum \frac{(a-b)^{2}}{(a+c)(b+c)}\leq \frac{\sum(a-b)^{2}}{ab+bc+ca}$

$\Leftrightarrow \sum \frac{c^{2}(a-b)^{2}}{(ab+bc+ca)(a+c)(b+c)}\geq 0$

Vậy bổ đề được chứng minh ta sẽ chứng minh 1 bất đẳng thức mạnh hơn
$(a^{2}+ab+b^{2})(b^{2}+bc+c^{2})(c^{2}+ca+a^{2})\geq \frac{1}{3}(2\sum a^{2}+\sum ab)(ab+bc+ca)^{2}$
$\Leftrightarrow 3(a^{2}+ab+b^{2})(b^{2}+ca+c^{2})(c^{2}+ca+a^{2})-(2\sum a^{2}+\sum ab)(ab+bc+ca)^{2}\geq 0$

$\Leftrightarrow \sum a^{2}b^{2}(a^{2}+b^{2})-abc[\sum a^{3}+\sum ab(a+b)+3abc]+2\sum a^{3}b^{3}\geq 0$
Ta chú ý các bất đẳng thức sau
$2\sum a^{3}b^{3}=\sum a^{3}(b^{3}+c^{3})\geq \sum a^{3}bc(b+c)=\sum abc^{3}(a+b)$
$\sum a^{2}b^{2}(a^{2}+b^{2})=\sum a^{4}(b^{2}+c^{2})\geq 2abc(a^{3}+b^{3}+c^{3})\geq abc(\sum a^{3}+3abc)$
Từ đây ta có điều cần chứng minh

 

Bài toán 16. (Võ Quốc Bá Cẩn,Trần Quang Hùng) Cho $a,b,c$ là các số thực không âm thỏa $a+b+c=1$. Chứng minh

$\dfrac{1}{\sqrt{(a^2+ab+b^2)(a^2+ac+c^2)}}+\dfrac{1}{\sqrt{(b^2+bc+c^2)(b^2+ba+a^2)}}+\dfrac{1}{\sqrt{(c^2+ca+a^2)(c^2+cb+b^2)}}\geq 4+\dfrac{8}{\sqrt{3}}$




#636292 VMF's Marathon Bất Đẳng Thức Olympic

Đã gửi bởi Gachdptrai12 on 28-05-2016 - 17:04 trong Bất đẳng thức và cực trị

Bài 13. (Sưu tầm) Cho $a,b,c>0$ thỏa $a+b+c=1$. Chứng minh

\[\frac{a}{b+c}+\frac{b}{a+c}+\frac{c}{a+b} \geq \sqrt{\frac{4-27abc}{4(ab+bc+ac)}}\]

Lời giải bài 13: Không mất tính tổng quát giả sử $a\geq b\geq c$, ta có một đẳng thức$(x+y+z)^{2}=3(xy+yz+zx)+\frac{1}{2}\sum (x-y)^{2}$

Từ đó ta có

BĐT$\Leftrightarrow (\frac{a}{b+c}+\frac{b}{c+a}+\frac{c}{a+b})^{2}\geq \frac{4(a+b+c)^{3}-27abc}{4(a+b+c)(ab+bc+ca)}$

Áp dụng đẳng thức trên ta có

$(\sum \frac{a}{b+c})^{2}=3\sum \frac{ab}{(a+c)(b+c)}+\frac{1}{2}\sum (\frac{a}{b+c}-\frac{b}{c+a})^{2}$

$=3\frac{\sum ab(a+b)}{(a+b)(b+c)(c+a)}+\frac{(a+b+c)^{2}}{2}\sum \frac{(a-b)^{2}}{(a+c)^{2}(b+c)^{2}}$

Mà $3 \frac{\sum ab(a+b)}{(a+b)(b+c)(c+a)}-\frac{9}{4}=\frac{3}{4}\sum \frac{c(a-b)^{2}}{(a+b)(b+c)(c+a)}$

Và $\frac{4(a+b+c)^{3}-27abc}{4(a+b+c)(ab+bc+ca)}-\frac{9}{4}=\frac{4(a^{3}+b^{3}+c^{3}-3abc)+3(a+b)(b+c)(c+a)-24abc}{4(a+b+c)(ab+bc+ca)}$

$= \sum \frac{(a-b)^{2}}{2(ab+bc+ca)}+\frac{3}{4}\sum \frac{c(a-b)^{2}}{(a+b+c)(ab+bc+ca)}\leq \sum \frac{(a-b)^{2}}{2(ab+bc+ca)}+\frac{3}{4}\sum \frac{c(a-b)^{2}}{(a+b)(b+c)(c+a)}$

Từ đó ta chỉ cần chứng minh được

$\sum (a-b)^{2}[\frac{(a+b+c)^{2}}{2(a+c)^{2}(b+c)^{2}}-\frac{1}{2(ab+bc+ca)}]\Leftrightarrow \sum S_{c}(a-b)^{2}\geq 0$

Dễ thấy $S_{c}\geq S_{b}\geq S_{a}$ ta có

$S_{a}+S_{b}=\frac{(a+b+c)^{2}}{2(a+b)^{2}(b+c^{2})}+\frac{(a+b+c)^{2}}{2(a+c)^{2}(b+c^{2})}-\frac{1}{ab+bc+ca}=\frac{(a+b+c)^{2}}{(a+b)^{2}(a+c)(b+c)}-\frac{1}{ab+bc+ca}\geq \frac{a+b+c}{(a+b)(b+c)(c+a)}-\frac{1}{ab+bc+ca}=\frac{abc}{(a+b+c)(ab+bc+ca)}\geq 0$

Từ đó ta có $S_{a}+S_{b}\geq 0$. Nếu $S_{a}\geq0$ thì bđt hiển nhiên đúng vì $S_{c}\geq S_{b}\geq S_{a} \geq 0$

Còn nếu $S_{a}\leq 0\Rightarrow S_{b}\geq 0$ (vì $S_{a}+S_{b}\geq 0$)

Ta chú ý tới bđt sau $S_{b}(a-c)^{2}\geq S_{b}(b-c)^{2}\Rightarrow S_{b}(a-c)^{2}+S_{a}(b-c)^{2}\geq (b-c)^{2}(S_{a}+S_{b})\geq 0$

Vậy ta có đpcm

 

Bài toán 14. (Micheal Rozenberg)

Cho $a,b,c$ dương thỏa $a+b+c=1$. Chứng minh

$\frac{a+b}{ab+1}+\frac{b+c}{bc+1}+\frac{c+a}{ca+1}\geq \frac{9}{5}$




#635999 VMF's Marathon Bất Đẳng Thức Olympic

Đã gửi bởi Gachdptrai12 on 27-05-2016 - 17:24 trong Bất đẳng thức và cực trị

Biến đổi theo $p,q,r$ ta được $: \frac{3r-2q+1}{q-r} \geq \sqrt{\frac{4-27r}{4q}} $
Áp dụng $: r \geq max \left \{ 0, \frac{4q-1}{9} \right \} (1)$
TH1 $: q \leq 0,25 \Rightarrow VT (1) \geq \frac{-2q+1}{q} $
Cần cm $: \frac{-2q+1}{q} \geq \sqrt{\frac{1}{q}} (2) $
Vì $q>0$ nên $: (2) \Leftrightarrow (q-1)(4q-1) \geq 0 (đúng ) $
TH2 $: q \geq 0,25 $
p\s $:$ Ai rảnh xét hộ e TH $2 $

 Bạn ơi bạn chưa xét đượ̣c hàm đó là đồng biến hay nghịch biến theo $r$ đâu mà cho vô ầm ầm thế




#635915 VMF's Marathon Bất Đẳng Thức Olympic

Đã gửi bởi Gachdptrai12 on 27-05-2016 - 12:38 trong Bất đẳng thức và cực trị

Bài toán 10. Cho $a, b, c >0$. Chứng minh rằng:
\[(a+b+c)\left(\dfrac{a}{b^2+c^2} + \dfrac{b}{c^2+a^2} + \dfrac{c}{a^2+b^2}\right) \geq 4+ \dfrac{4a^2b^2c^2}{(a^2+b^2)(b^2+c^2)(c^2+a^2)}\]

 Lời giải 3 cho bài 10:
 Viết bài toán dưới dạng $pqr$ là $p(11p^{2}-21q)r+(p^{2}-4q)(p^{2}-2q)(p^2+q)\geq0$
 Dễ thấy hệ số cuả $r$ là số dương nên theo phương pháp ABC thì cực trị đạt được tại 2 biến bằng nhau và một biến bằng $0$
 TH1: $b=c$ và do tính đồng bậc ta có thể giả sử $b=1$, do đó bất đẳng thức ban đầu trở thành
$(a+2)(\frac{a}{2}+\frac{2}{a^{2}+1}\geq 4+\frac{4a^2}{2(a^{2}+1)^{2}}$
$\Leftrightarrow a(a-1)^{2}(4a^{3}+4a^{2}+a+6)\geq0$ (hiển nhiên đúng)
 TH2: $c=0$ bất đẳng thức ban đầu trở thành
$(a+b)(\frac{a}{b^{2}}+\frac{b}{a^{2}})\geq4$
Tương đương $(a+b)(a^{3}+b^{3})\geq4a^{2}b^{2}$ (hiển nhiên đúng theo AM-GM)
 Vậy bất đẳng thức được chứng minh

 

PS




#635882 VMF's Marathon Bất Đẳng Thức Olympic

Đã gửi bởi Gachdptrai12 on 27-05-2016 - 10:06 trong Bất đẳng thức và cực trị

Bài toán 12. (Phan Hồng Sơn) Cho $a,b,c>0$ thỏa $a+b+c=3$. Chứng minh rằng

$\frac{a^{2}}{\sqrt{bc+(a-1)^{2}}}+\frac{b^{2}}{\sqrt{ca+(b-1)^{2}}}+\frac{c^{2}}{\sqrt{ab+(c-1)^{2}}}\geq 3$




#635456 VMF's Marathon Bất Đẳng Thức Olympic

Đã gửi bởi Gachdptrai12 on 25-05-2016 - 16:04 trong Bất đẳng thức và cực trị

Bài toán 8. Cho $a,b,c$ là các số thực dương. Chứng minh

\[\frac{\sqrt{a+b}}{c}+\frac{\sqrt{b+c}}{a}+\frac{\sqrt{a+c}}{b}\geq 3\sqrt{\frac{2(a+b+c)}{ab+bc+ca}}\]




#635455 VMF's Marathon Bất Đẳng Thức Olympic

Đã gửi bởi Gachdptrai12 on 25-05-2016 - 15:59 trong Bất đẳng thức và cực trị

 

 Bài toán 7. (VMF) Cho $a,b,c\geq 0$ trong đó không có hai số nào đồng thời bằng 0 và số thực $k$ thoả mãn $3^k\geq 2^{k+1}$. Chứng minh rằng : 

\[\frac{1}{a^k+b^k}+\frac{1}{b^k+c^k}+\frac{1}{c^k+a^k}\geq \frac{5.2^{k-1}}{(a+b+c)^k}\]

 

 Lời giải bài 7: Ta có bổ đề $\frac{2}{x^{k}+y^{k}}+\frac{1}{x^{k}}+\frac{1}{y^{k}}\geq \frac{3.2^{k}}{(x+y)^{k}}$

 Chứng minh áp dụng AM-GM ta có 

$\frac{2}{x^{k}+y^{k}}+\frac{x^{k}+y^{k}}{2x^{k}y^{k}}+\frac{x^{k}+y^{k}}{2x^{k}y^{k}}\geq \frac{2}{x^{k}+y^{k}}+\frac{x^{k}+y^{k}}{2x^{k}y^{k}}+\frac{1}{x^{\frac{k}{2}}y^{\frac{k}{2}}}\geq 3\sqrt[3]{\frac{1}{(xy)\frac{3k}{2}}}\geq \frac{3.2^{k}}{(x+y)^{k}}$

Đúng, vậy bổ đề được chứng minh

Ta có giả thiết được viết lại là $k\geq \log_{\frac{3}{2}}2$ 

Không mất tính tổng quát giả sử $a\geq b\geq c$ ta có các nhận xét

$(a+\frac{c}{2})^{k}\geq a^{k}+c^{k}$

$(b+\frac{c}{2})^{k}\geq b^{k}+c^{k}$

Thật vậy với c=0 thì các nhận xét hiển nhiên đúng với c khác 0 ta có bất đẳng thức trên tương đương 
$(t+\frac{1}{2})^{k}-t^{k}-1\geq 0$ và $t=\frac{a}{c}\geq 1$
Xét hàm $f(t)=(t+\frac{1}{2})^{k}-t^{k}-1$ với $t\in [1;+\infty )\Rightarrow f'(t)=k[(t+\frac{1}{2})^{k-1}-t^{k-1}]>0$
Vậy $f(t)$ đồng biến trên$[1;+\infty )$
Vậy $f(t)\geq f(1)=(\frac{3}{2})^{k}-2\geq (\frac{3}{2})^{\log_{\frac{3}{2}^{2}}}-2=0$ vậy nhận xét đc chứng minh
Vậy $\sum \frac{1}{(a+\frac{c}{2})^{k}}\geq \frac{1}{(a+\frac{c}{2})^{k}}+\frac{1}{(b+\frac{c}{2})^{k}}+\frac{1}{a^{k}+b^{k}}\geq \frac{3.2^{k}}{(a+b+c)^{k}}-\frac{2}{(a+\frac{c}{2})^{k}+(b+\frac{c}{2})^{k}}+\frac{1}{a^{k}+b^{k}}$
Ta cần chứng minh
$\frac{3.2^{k}}{(a+b+c)^{k}}-\frac{2}{(a+\frac{c}{2})^{k}+(b+\frac{c}{2})^{k}}+\frac{1}{a^{k}+b^{k}}\geq \frac{5.2^{k-1}}{(a+b+c)^{k}}$
$\Leftrightarrow \frac{2^{^{k-1}}}{(a+b+c)^{k}}+\frac{1}{a^{k}+b^{k}}\geq \frac{2}{(a+\frac{c}{2})^{k}+(b+\frac{c}{2})^{k}}$
Theo bất đẳng thức holder ta có
$2^{k-1}[(a+\frac{c}{2})^{k}+(b+\frac{c}{2})^{k}]\geq (a+b+c)^{k}$
mà hiển nhiên ta có$(a+\frac{c}{2})^{k}+(b+\frac{c}{2})^{k}\geq (a+b)^{k}$
Vậy ta có đpcm 
 
 \begin{array}{| l | l |} \hline \text{HDTterence2k} & 1\\ \hline \text{hoanglong2k} & 3\\ \hline \text{Gachdptrai12} & 4\\ \hline \text{Nguyenhuyen_AG} & 3\\ \hline \text{fatcat12345} & 1\\ \hline \text{lenhatsinh3} & 1\\ \hline\end{array} 



#635444 VMF's Marathon Bất Đẳng Thức Olympic

Đã gửi bởi Gachdptrai12 on 25-05-2016 - 15:23 trong Bất đẳng thức và cực trị

Bài toán đề nghị
Bài toán 6. Cho $a,b,c$ là các số thực thỏa mãn $ab+bc+ca=3$. Chứng minh
\[4(a^4+b^4+c^4)+11abc(a+b+c)\ge 45.\]

Viết lại bất đẳng thức về dạng thuần nhất
$4(a^4+b^4+c^4)+11abc(a+b+c)\geq 5(ab+bc+ca)^2$
$\Leftrightarrow 4(a^4+b^4+c^4)+abc(a+b+c)\geq 5(a^2b^2+b^2c^2+c^2a^2)$ (1)
Vì bđt này thuần nhất nên ta có thể bỏ điều kiện $ab+bc+ca=3$ và ta có thể chuẩn hóa $a+b+c=3$
Đặt $x+1=a,y+1=b,z+1=c$ từ đó ta có $x+y+z=0$
Thay $z=-x-y$ vào (1) ta có
$4(x^{4}+y^{4}+z^{4}+4(x^{3}+y^{3}+z^{3})+6(x^{2}+y^{2}+z^{2})+3)+ 33(xyz+xy+yz+zx+1)\geq 5(xy+yz+zx+3)^{2}$
$\Leftrightarrow 4[2(x^{2}+xy+y^{2})^{2}-12xy(x+y)+12(x^{2}+xy+y^{2})+3]+33[-xy(x+y)-(x^{2}+xy+y^{3})+1]\geq 5[3-(x^{2}+xy+y^{2})]^{2}$
$\Leftrightarrow 3(x^{2}+xy+y^{2})^{2}+45(x^{2}+xy+y^{2})\geq 81xy(x+y)$
Ta có các nhận xét sau $(x^{2}+xy+y^{2})\geq \frac{3}{4}(x+y)^{2}\geq 3xy$ nhưng bất dẳng thức đúng nếu $xy\geq0$ mà ta có $xy.xy.zx=(xyz)^2\geq0$ nên tồn tại 1 trong 3 số có 1 số lớn hơn 0, giả sử đó là xy (thỏa điểu kiện của nhận xét)
Nên ta có
$3(x^{2}+xy+y^{2})^{2}+45(x^{2}+xy+y^{2})\geq 27x^{2}y^{2}+\frac{135}{4}(x+y)^{2}$
Bây giờ ta cm
$3(x^{2}+xy+y^{2})^{2}+45(x^{2}+xy+y^{2})\geq 27x^{2}y^{2}+\frac{135}{4}(x+y)^{2}\geq81xy(x+y)$ bất đẳng thức này dễ chứng minh



#635420 VMF's Marathon Bất Đẳng Thức Olympic

Đã gửi bởi Gachdptrai12 on 25-05-2016 - 13:34 trong Bất đẳng thức và cực trị

 

 

 

 

 Bài toán 5. (Phạm Kim Hùng) Cho các số thực $a,b,c$ dương. Tìm hằng số $k$ tốt nhất sao cho bất đẳng thức sau luôn đúng

\[\dfrac{a^3+b^3+c^3}{(a+b)(b+c)(c+a)}+\dfrac{k(ab+bc+ca)}{(a+b+c)^2}\geq \dfrac{3}{8}+\dfrac{k}{3}\]

 

Lời giải khác :

Cho $b=a=1+\sqrt{3},c=1$ ta có $k\leq \frac{9(3+2\sqrt{3})}{8}$

Ta chứng minh $k_{0}=\frac{9(3+2\sqrt{3})}{8}$ là giá trị cần tìm

Thật vậy ta sẽ chứng minh

$\dfrac{a^3+b^3+c^3}{(a+b)(b+c)(c+a)}+\dfrac{k_{0}(ab+bc+ca)}{(a+b+c)^2}\geq \dfrac{3}{8}+\dfrac{k_{0}}{3}$

Vì bđt của chúng ta thuần nhất nên ta chuẩn hóa$ a+b+c=1$ và đặt $r=abc$ và $ab+bc+ca=\frac{1-q^{2}}{3}$

Ta có bđt trở thành $\frac{3(3r+q^{2})}{-3r+1-q^{2}}+\frac{k_{0}(1-q^{2})}{3}\geq \frac{3}{8}+\frac{k_{0}}{3}$

Và đây là hàm đồng biến theo r nên nếu $2q\geq 1$ thì

$VT\geq \frac{3q^{2}}{1-q^{2}}+\frac{k_{0}(1-q^{2})}{3}\geq 1+\frac{k_{0}}{4}> \frac{3}{8}+\frac{k_{0}}{3}$

(Vì là hàm đồng biến theo $q^{2}\geq 1/4$)

Nếu $2q \leq 1$ thì ta áp dụng bổ đề của VQBC $\frac{(1+q)^{2}(1-2q)}{27}\leq r\leq \frac{(1-2q)^{2}(1+2q)}{27}$

Ta chứng minh $\frac{3((1+q)^{2}(1-2q)+9q^{2})}{-(1+q)^{2}(1-2q)+9(1-q^{2})}+\frac{k_{0}(1-q^{2})}{3}\geq \frac{3}{8}+\frac{k_{0}}{3}$

Từ đây ta phân tích được $VT-VP=\frac{3q^{2}(3+2\sqrt{3})(2\sqrt{3}-1-q)(q-2+\sqrt{3})}{8(q+1)(q-2)^{2}}\geq 0$

Hiển nhiên đúng, vậy $k_{max}=\frac{9(3+2\sqrt{3})}{8}$

p/s anh Huyện nhanh tay quá  :ohmy:  :ohmy:

 

\begin{array}{| l | l |} \hline \text{HDTterence2k} & 1\\ \hline \text{hoanglong2k} & 2\\ \hline \text{Gachdptrai12} & 2\\ \hline \text{Nguyenhuyen_AG} & 3\\ \hline \text{fatcat12345} & 1\\ \hline \text{lenhatsinh3} & 1\\ \hline\end{array} 




#635320 VMF's Marathon Bất Đẳng Thức Olympic

Đã gửi bởi Gachdptrai12 on 24-05-2016 - 22:52 trong Bất đẳng thức và cực trị

Cách 2 cho bài 2:

Đặt $x=\sqrt{a+b}$ và $y,z$ tương tự, từ đó ta có $a=\frac{x^{2}+z^{2}-y^{2}}{2}$ và cũng tương tự cho $b,c$

Từ đó ta có

$\left | P \right |=\left | \sum \frac{a-b}{\sqrt{a+b}} \right |$

$=\left | \sum \frac{(x^{2}+z^{2}-y^{2})-(x^{2}+y^{2}-z^{2})}{2x} \right |$

$=\left |\sum \frac{(z^{2}-x^{2})}{x} \right |=\left | \sum \frac{(x-y)(y-z)(z-x)(x+y+z)}{xyz} \right |$

$=\left | \frac{(x^{2}-y^{2})(y^{2}-z^{2})(z^{2}-x^{2})}{xyz(x+y)(y+z)(z+x)} \right |$

$=\left | \frac{(a-b)(b-c)(c-a)(\sqrt{a+b}+\sqrt{b+c}+\sqrt{c+a})}{\sqrt{(a+b)(b+c)(c+a)}(\sqrt{a+b}+\sqrt{a+c})(\sqrt{b+c}+\sqrt{c+a})(\sqrt{b+c}+\sqrt{b+a})} \right |$

Đặt $c= \min \{a,b,c\}$, ta có các nhận xét 

 - $\left | (a-b)(b-c)(c-a) \right |\leq \left | ab(a+b) \right |$

 - $\frac{1}{\sqrt{(a+b)(b+c)(c+a)}}\leq \frac{1}{ab(a+b)}$

 - $\frac{1}{(\sqrt{a+b}+\sqrt{a+c})(\sqrt{b+c}+\sqrt{b+a})}\leq \frac{1}{(\sqrt{a}+\sqrt{a+b})(\sqrt{b}+\sqrt{a+b})}$

 - $\frac{\sqrt{a+b}+\sqrt{b+c}+\sqrt{c+a}}{\sqrt{b+c}+\sqrt{c+a}}= 1+\frac{\sqrt{a+b}}{\sqrt{a+c}+\sqrt{b+c}}\leq 1+ \frac{\sqrt{a+b}}{\sqrt{a}+\sqrt{b}}$

Từ các bất đẳng thức trên ta có

$\left | P \right |\leq \frac{\left | (ab(a-b)) \right |(\sqrt{a}+\sqrt{b}+\sqrt{a+b})}{\sqrt{ab(a+b)}(\sqrt{a}+\sqrt{a+b})(\sqrt{b}+\sqrt{a+b})}$

$=\left | \frac{a-b}{\sqrt{a+b}}+\sqrt{b}-\sqrt{a} \right |=\frac{1}{\sqrt{a+b+c}}\left | \frac{a-b}{\sqrt{a+b}}+\sqrt{b}-\sqrt{a} \right |$

$\leq \frac{1}{\sqrt{a+b}}\left | \frac{a-b}{\sqrt{a+b}}+\sqrt{b}-\sqrt{a} \right |=\left | \frac{a-b}{a+b}+\frac{\sqrt{b}-\sqrt{a}}{\sqrt{a+b}} \right |$

Đặt $x= \frac{a}{a+b}\Rightarrow \frac{b}{b+a}=1-x$

Suy ra $\frac{a-b}{a+b}+\frac{\sqrt{a}-\sqrt{b}}{\sqrt{a+b}}=x-(1-x)+\sqrt{1-x}-\sqrt{x}=2x-\sqrt{x}+\sqrt{1-x}-1=f(x)$

           $\Rightarrow f'(x)=2-\frac{1}{2\sqrt{x}}-\frac{1}{2\sqrt{1-x}}$

Giải phương trình $f'(x)=0$ ta có 2 nghiệm $x_{1}$ và $x_{2}$  là $\frac{8\pm \sqrt{46-2\sqrt{17}}}{16}$ và ta có $f(x_{1})\geq f(x)\geq f(x_{2})$

 Từ đó tìm được $\min$ và $\max$ của $P$




#634754 tài liệu BW

Đã gửi bởi Gachdptrai12 on 22-05-2016 - 18:42 trong Tài liệu, chuyên đề, phương pháp về Bất đẳng thức

tìm tài liệu về phương pháp BW tiếng anh lẫn tiếng việt đều được 




#634672 $\sum_{cyc}{\frac{(a+b)(b+c)}{b^...

Đã gửi bởi Gachdptrai12 on 22-05-2016 - 11:49 trong Bất đẳng thức - Cực trị

cho $a, b, c>0, a+b+c=3$. chứng minh


$\sum_{cyc}{\frac{(a+b)(b+c)}{b^3+c^3+abc}}+\frac{1}{4}\ge\frac{a^2+b^2+c^2+48abc}{4(ab+bc+ca)}$




#634668 $(a-b)^2(6a^4-b^4)+(b-c)^2(6b^4-c^4)+(c-a)^2(6c^4-a^4)\geq0$

Đã gửi bởi Gachdptrai12 on 22-05-2016 - 11:23 trong Bất đẳng thức - Cực trị

 cho $a,b,c$ là các số thực chứng minh

$(a-b)^2(6a^4-b^4)+(b-c)^2(6b^4-c^4)+(c-a)^2(6c^4-a^4)\geq0$




#634313 Chứng minh đường thẳng qua $A$ vuông góc $TD$ đi qua...

Đã gửi bởi Gachdptrai12 on 20-05-2016 - 18:43 trong Hình học

Cho tam giác $ABC$ nhọn nội tiếp $\odot (O)$ với $B,C$ cố định. Tiếp tuyến tại $A$ của $\odot (O)$ cắt $BC$ tại $T. M$ là trung điểm $BC$. Đường thẳng qua $T$ vuông góc $AM$ cắt $CA,AB$ tại $E,F.BE$ cắt $CF$ tại $D$.Chứng minh đường thẳng qua $A$ vuông góc $TD$ đi qua điểm cố đinh.




#634046 3 - Mở rộng: Đánh giá từng biến

Đã gửi bởi Gachdptrai12 on 19-05-2016 - 09:47 trong Bất đẳng thức - Cực trị

 

 Bài toán 7:

 
 
Cho x, y và z là ba số thực thuộc đoạn $\left [ 1; 3 \right ]$, thoả mãn:
 
$x+y+z=6.$
 
 Hãy tìm giá trị lớn nhất của biểu thức:
 
$P=\frac{x^{2}y^{2}+y^{2}z^{2}+z^{2}x^{2}+12xyz+72}{xy+yz+zx}-\frac{1}{2}xyz.$
 
 
 P/S:
 
 Có thành viên nào có hướng tiếp cận cho 2 bài toán 3 và 6 chưa ạ?

 

Bài 7 là đề thi ĐH năm 2015 bài này đặt về t=ab+bc+ca và từ đk ta tìm được [11;12] và hàm theo f(t) là hàm đồng biến  :icon6:




#633223 $\sum \frac{1}{\sqrt{x^{2}-...

Đã gửi bởi Gachdptrai12 on 15-05-2016 - 10:40 trong Bất đẳng thức - Cực trị

cho x,y,z là các số thực ko âm thỏa x+y+z=1  tìm min 

$\frac{1}{\sqrt{x^{2}-xy+y^{2}}}+\frac{1}{\sqrt{y^{2}-yz+z^{2}}}+\frac{1}{\sqrt{z^{2}-zx+x^{2}}}$